LSAT and Law School Admissions Forum

Get expert LSAT preparation and law school admissions advice from PowerScore Test Preparation.

User avatar
 Dave Killoran
PowerScore Staff
  • PowerScore Staff
  • Posts: 5852
  • Joined: Mar 25, 2011
|
#41327
Complete Question Explanation
(The complete setup for this game can be found here: lsat/viewtopic.php?t=8467)

The correct answer choice is (C)

Do not forget to convert Must Be False into Cannot Be True! This question uses the same Hurdle the UncertaintiesTM principle seen in question #9. If M lives on the fourth floor, then the MN block completely occupies the fourth floor, again creating a wall in the interior of the game. This affects the placement of the KP block, which is now limited to floors one and two (1-2) or floors two and three (2-3). Accordingly, either K or P must live on the second floor:

pt2_o91_g2_q10.png

Thus, no other variable can live on the second floor, and answer choice (C) must be correct.
You do not have the required permissions to view the files attached to this post.
 ashley
  • Posts: 3
  • Joined: Mar 24, 2011
|
#153
I am looking at the Logic Games Bible, page 73, question ten, and I believe that not only is C a correct answer, but A is also a correct answer.

A.) is correct because if O lives on the fifth floor, and MN take up the fourth floor, then Q can only live on the 3rd floor and k and P have to live on 2 and 1 respectively. There is now no place for L to live. L can't live with another letter (s/he lives in the only apartment on her/his floor).

Is this correct?
User avatar
 Dave Killoran
PowerScore Staff
  • PowerScore Staff
  • Posts: 5852
  • Joined: Mar 25, 2011
|
#162
Hi Ashley,

Thanks for the message. Let's look at the question you raised.

In question #10, it appears to me that you have assumed that O must live on a floor with only one apartment. In the scenario you describe, Q can live on the third floor, but then K can also live on the third floor, leaving P on the second floor and then L alone on the first floor, like so:

5. O J
4. M N
3. K Q
2. P
1. L

That scenario satisfies all of the rules and also shows that O can live on the fourth floor.

I hope the above helps explain the problem, and please do not hesitate to let me know if you have any other questions. Thanks!
User avatar
 Dave Killoran
PowerScore Staff
  • PowerScore Staff
  • Posts: 5852
  • Joined: Mar 25, 2011
|
#19596
Hi,

This explanation is posted in response to a student question about the game on page 173 of the LSAT Logic Games Bible.

#10 can be tricky, but it does not contain an error. Let’s take a look at it more closely and break it down step-by-step:

The question stem for #10 specifies that M lives on the fourth floor. So, from the fourth rule, N must also live on the fourth floor:

  • 5
    4 M N
    3
    2
    1
As elaborated elsewhere in the game explanation, that means that the KP vertical block must go on floors 3-2 or 2-1; in other words, K or P must live on the second floor:

  • 5
    4 M N
    3
    2 K/P
    1
From the third rule, we know that the second floor has only one apartment, and so the second floor will be made up entirely of either K or P but not both). This creates a situation where the second and fourth floors are filled—no other variables can be placed on either floor, and the remaining variables—J, L, O, Q, and remainder of K/P mist be placed on the first, third, and fifth floors. I feel that you probably reached this point without too much trouble, so the issue comes once we get into the rest of the questions stem. So, as noted in the book, this is a Cannot Be False question, which is the same thing as Cannot Be True. So you are looking for an answer that cannot happen.

I think answer choice (C) made sense to you, so I’ll focus on (A). Answer choice (A) states that O lives on the fifth floor. If that is impossible, it will be a correct answer; if it is possible, then it will be incorrect. So let’s take a look, and let’s start by putting O on the fifth floor:

  • 5 O
    4 M N
    3
    2 K/P
    1
From the fifth rule, we know that O and Q cannot live on the same floor, and so Q cannot live on the fifth floor, leaving only the first and third floors as options for Q. But, the last rule states that Q cannot live on the first or second floor, and as a result we can conclude that Q must live on the third floor:

  • 5 O
    4 M N
    3 Q
    2 K/P
    1
We still have KL floating around, but from the sixth rule we know that L lives in the only apartment on her floor. The only floor that is currently unoccupied is the first floor, and so there is no other choice but to put L on the first floor:

  • 5 O
    4 M N
    3 Q
    2 K/P
    1 L
Of course, L must live alone on her floor, so that impacts the K and P block, and we can conclude that P can’t live on the first floor. That means that P lives on the second floor, and consequently that K lives on the third floor:

  • 5 O
    4 M N
    3 Q K
    2 P
    1 L
There’s only one variable left, and that is J. The only rule about J is that J lives on a floor with two apartments. There are only three floors that currently have one resident: the first, second, and fifth floors. But the fist floor is occupied by L, who must be alone, so the first floor is out. And the second floor has only one apartment per the third rule, so the second floor is out. That leaves just the fifth floor available. Van J live on the fifth floor? Yes, there’s no restriction there, resulting in the following diagram:

  • 5 O J
    4 M N
    3 Q K
    2 P
    1 L
Consequently, it is possible for O to live on the fifth floor and to have a viable solution. So, answer choice (A) can occur, and thus is incorrect.


Overall, a tough question, but one that does not have an error. You’ll find this happens sometimes—you look at a question for so long that there just doesn’t appear to be a solution, and it looks like the question is flawed. Flawed questions are incredibly rare on the LSAT, so if you run into something like this, just choose one of the two contending answers and move on. Don’t try to figure it out; you can look at it again when the test is released (or if it’s a practice test, at the end :-D ).

Please let me know if the above helps out, and if you have any other questions, just let me know. Thanks!
 Echx73
  • Posts: 36
  • Joined: Nov 11, 2015
|
#20615
Team PowerScore,

When I came upon Question 10 on page 173, I attacked the question in the following manner....

#1 I saw the 'if,' so we know this is going to be a local game.

#2 I drew out the five floors and I applied the new rule along with the existing rules given in the game summary.

#3 When I went to apply the rule "K lives on the floor directly above P" I noticed the KP block can go on floor 3 & 2 or 2 & 1. When I saw this, immediately drew another building right next to my original structure. One Building has KP block on floors 3 & 2 and my second drawing has KP block on floors 2 & 1.

#4 (I have my game structure setup properly now it is time to work) I know the correct answer is going to be false, and the wrong answers are going to be possibly true or true. My first inclination when I looked at the answers was to start at answer A- test the answer to see the viability and if it was not false- I would work down the list deducting and crossing out each answer until I found the correct one. The correct answer for this question is 'C'

My question is: How would you have attacked this question? Should my first inclination not to test each answers, but to skim through each answer to see if something immediately pops out?

I appreciate your help! Thank you for your time.

Sincerely,

Eric
User avatar
 Dave Killoran
PowerScore Staff
  • PowerScore Staff
  • Posts: 5852
  • Joined: Mar 25, 2011
|
#20617
Hi Eric,

This is a really good question. Let me use your steps and show you where I would have gone a different direction. I'll put my comments in blue:

  • #1 I saw the 'if,' so we know this is going to be a local game. Yep, exactly the same.

    #2 I drew out the five floors and I applied the new rule along with the existing rules given in the game summary. I would have waited a second before drawing out the five floors. What I would have done first is look at the rule being added, and see if I saw any immediate implications, or anything of interest to me (something unusual, or some chain reaction of sorts). M on the fourth floor blocks that floor off entirely, and so I'd then mentally go right to the KP block since I know it will be affected by a placement such as this. If I didn't see it immediately, then I'd draw out the five floors.

    #3 When I went to apply the rule "K lives on the floor directly above P" I noticed the KP block can go on floor 3 & 2 or 2 & 1. When I saw this, immediately drew another building right next to my original structure. One Building has KP block on floors 3 & 2 and my second drawing has KP block on floors 2 & 1. Excellent recognition on the placement of KP! From there, you could go either way here: drawing the two diagrams works great, as does putting the K/P dual-option on the second floor in a single diagram.

    #4 (I have my game structure setup properly now it is time to work) I know the correct answer is going to be false, and the wrong answers are going to be possibly true or true. My first inclination when I looked at the answers was to start at answer A- test the answer to see the viability and if it was not false- I would work down the list deducting and crossing out each answer until I found the correct one. The correct answer for this question is 'C' This is where we separate a little bit, in part because I like to avoid doing work until I feel it's absolutely necessary :-D So, I'd say that in this instance, you should make a quick skim-through before working. Here's how that would work:

    Once I've drawn out the mini-diagram(s) next to the problem, there's a mental step here that is critical. We've seen that KP is 3-2 or 2-1. So, I know the second floor is occupied. The question stem wants to know what Cannot Be True. While it is the case that the incorrect answers Could Be True or Must Be True, here I know that something is always the case (K/P on 2), and so I know that anything else to do with the second floor cannot occur. So I go looking for it, and that leads me right to answer choice (C). The principle here is that when you have a Cannot question, anything that must always happen can help you because it stops other variables from being there (they cannot occur); conversely, if you have a Must question, anything that cannot occur helps you out because those things must always not occur. Thus, once I have an inference that results in something that must be true (and something that isn't easy to spot right off the bat), there's a decent chance they will make a play on that.

    Part of how I see this comes from the K/P variation on the second floor. The fact that the KP block is limited to 3-2 or 2-1 is what I call the first-level step from the placement of M. It's a direct implication. In LG questions that are above the easy level of difficulty, first level implications are not normally the correct answer. The second-level implication is that the second floor is now occupied entirely by K or P, and once I hit a second-level implication, I start thinking we might be looking at something that relates to the correct answer. This feeling is strengthened by the presence of the Cannot form question stem, which adds a layer of difficulty to the problem.

    This process probably sounds tricky, but once you've seen enough games, you get used to how they do this, and it becomes much easier to see these ideas in the answer choices.
Please let me know if that helps. Thanks!

Get the most out of your LSAT Prep Plus subscription.

Analyze and track your performance with our Testing and Analytics Package.